You are on page 1of 45

GATE EC

2005

Question 1 - 30 Carry One Mark Each


MCQ 1.1

The following differential equation has


d2 y
dy 3
3 c 2 m + 4 c m + y2 + 2 = x
dt
dt
(A) degree = 2 , order = 1

(B) degree = 1, order = 2

(C) degree = 4 , order = 3

(D) degree = 2 , order = 3

SOL 1.1

Order is the highest derivative term present in the equation and degree is the power
of highest derivative term.
Order = 2 , degree = 1
Hence (B) is correct answer.

MCQ 1.2

Choose the function f (t); 3 < t < 3 for which a Fourier series cannot be defined.
(A) 3 sin (25t)
(B) 4 cos (20t + 3) + 2 sin (710t)
(C) exp ( t ) sin (25t)

(D) 1

SOL 1.2

Fourier series is defined for periodic function and constant.


3 sin (25t) is a periodic function.
4 cos (20t + 3) + 2 sin (710t) is sum of two periodic function and also a periodic
function.
e t sin (25t) is not a periodic function, so FS cant be defined for it.
1 is constant
Hence (C) is correct option.

MCQ 1.3

A fair dice is rolled twice. The probability that an odd number will follow an even
number is
(A) 1/2
(B) 1/6
(C) 1/3

SOL 1.3

(D) 1/4

Probability of coming odd number is 12 and the probability of coming even number
is 12 . Both the events are independent to each other, thus probability of coming odd
number after an even number is 12 # 12 = 14 .
Hence (D) is correct answer.

Brought to you by: Nodia and Company


PUBLISHING FOR GATE

Visit us at: www.nodia.co.in

Page 2

MCQ 1.4

SOL 1.4

GATE EC 2005

www.gatehelp.com

A solution of the following differential equation is given by


(A) y = e2x + e3x

(B) y = e2x + e3x

(C) y = e2x + 33x

(D) y = e2x + e3x

d2 y
dy
5
+ 6y = 0
2
dx
dx

Hence (B) is correct answer.


d2 y
dy
We have
5
+ 6y = 0
2
dx
dx
m2 5m + 6 = 0
m = 3, 2
The CF is
yc = C1 e3x + C2 e2x
Since Q = 0 , thus
y = C1 e3x + C2 e2x
Thus only (B) may be correct.
The A.E. is

MCQ 1.5

SOL 1.5

The function x (t) is shown in the figure. Even and odd parts of a unit step function
u (t) are respectively,

(A) 1 , 1 x (t)
2 2

(B) 1 , 1 x (t)
2 2

(C) 1 , 1 x (t)
2
2
Hence (A) is correct answer.
g (t) + g ( t)
Ev{g (t)} =
2
g (t) g ( t)
odd{g (t)} =
2

(D) 1 , 1 x (t)
2
2

Here
Thus

MCQ 1.6

g (t) = u (t)
u (t) + u ( t)
ue (t) =
=
2
u (t) u ( t)
uo (t) =
=
2

1
2
x (t)
2

The region of convergence of z transform of the sequence


5 n
6 n
b 6 l u (n) b 5 l u ( n 1) must be
(A) z < 5
(B) z > 5
6
6

Brought to you by: Nodia and Company


PUBLISHING FOR GATE

Visit us at: www.nodia.co.in

Page 3

SOL 1.6

GATE EC 2005

(C) 5 < z < 6


6
5
Hence (C) is correct answer.
Here
x1 (n) = ` 5 jn u (n)
6
1
X1 (z) =
1 ^ 65 z1h
x2 (n) =` 6 jn u ( n 1)
5
1
X1 (z) = 1
1 ^ 65 z1h

www.gatehelp.com

(D) 6 < z < 3


5

ROC : R1 " z > 5


6

ROC : R2 " z < 6


5

Thus ROC of x1 (n) + x2 (n) is R1 + R2 which is 5 < z < 6


6
5
MCQ 1.7

The condition on R, L and C such that the step response y (t) in the figure has no
oscillations, is

(A) R $ 1
2

L
C

L
C
Transfer function is
(C) R $ 2

SOL 1.7

(B) R $

L
C

(D) R =

1
LC

1
1
Y (s)
1
sC
LC
=
= 2
=
2
1
R
U (s)
s
LC
scR
1
+
+
R + sL +
s + s+ 1
sC
L
LC
2
2
Comparing with s + 2n s + n = 0 we have
Here
2n = R ,
L
and
n = 1
LC
Thus
= R LC = R C
2L
2
L
For no oscillations, $ 1
Thus R C $ 1
2
L
or
R $2 L
C
Hence (C) is correct option.
Brought to you by: Nodia and Company
PUBLISHING FOR GATE

Visit us at: www.nodia.co.in

Page 4
MCQ 1.8

GATE EC 2005

www.gatehelp.com

The ABCD parameters of an ideal n: 1 transformer shown in the figure are


n 0
>0 x H

The value of x will be

SOL 1.8

(A) n

(B) 1
n

(C) n2

(D) 12
n

For given transformer


I2 = V1 = n
I1
V2
1
or
I1 = I2 and V1 = nV2
n
Comparing with standard equation
V1 = AV2 + BI2
I1 = CV2 + DI2
A B
n 0
=C D G = = 0 1 G
n
Thus
x = 1
n
Hence (B) is correct option.

MCQ 1.9

In a series RLC circuit, R = 2 k , L = 1 H, and C = 1 F The resonant frequency


400
is
(B) 1 # 10 4 Hz
(A) 2 # 10 4 Hz

(D) 2 # 10 4 Hz

(C) 10 4 Hz
SOL 1.9

Hence (B) is correct option.


We have L = 1H and C = 1 # 106
400
Resonant frequency
1
f0 =
==
2 LC
2

1
1 # 1 # 10 - 6
400

3
4
= 10 # 20 = 10 Hz
2

Brought to you by: Nodia and Company


PUBLISHING FOR GATE

Visit us at: www.nodia.co.in

Page 5
MCQ 1.10

SOL 1.10

GATE EC 2005

www.gatehelp.com

The maximum power that can be transferred to the load resistor RL from the
voltage source in the figure is

(A) 1 W

(B) 10 W

(C) 0.25 W

(D) 0.5 W

Maximum power will be transferred when RL = Rs = 100


In this case voltage across RL is 5 V, therefore
2
Pmax = V = 5 # 5 = 0.25 W
R
100
Hence (C) is correct option.

MCQ 1.11

The bandgap of Silicon at room temperature is


(A) 1.3 eV
(B) 0.7 eV
(C) 1.1 eV

(D) 1.4 eV

SOL 1.11

For silicon at 0 K,
Eg0 = 1.21 eV
At any temperature
EgT = Eg0 3.6 # 10 - 4 T
At T = 300 K,
Eg300 = 1.21 3.6 # 10 - 4 # 300 = 1.1 eV
This is standard value, that must be remembered.
Hence option (C) is correct.

MCQ 1.12

A Silicon PN junction at a temperature of 20c C has a reverse saturation current


of 10 pico - Ameres (pA). The reserve saturation current at 40cC for the same bias
is approximately
(A) 30 pA
(B) 40 pA
(C) 50 pA

SOL 1.12

(D) 60 pA

The reverse saturation current doubles for every 10cC rise in temperature as follows
I0 (T) = I 01 # 2(T T )/10
Thus at 40c C, I0 = 40 pA
Hence option (B) is correct.
1

MCQ 1.13

The primary reason for the widespread use of Silicon in semiconductor device
technology is

Brought to you by: Nodia and Company


PUBLISHING FOR GATE

Visit us at: www.nodia.co.in

Page 6

GATE EC 2005

www.gatehelp.com

(A) abundance of Silicon on the surface of the Earth.


(B) larger bandgap of Silicon in comparison to Germanium.
(C) favorable properties of Silicon - dioxide (SiO2)
(D) lower melting point
SOL 1.13

Silicon is abundant on the surface of earth in the from of SiO2 .


Hence option (A) is correct.

MCQ 1.14

The effect of current shunt feedback in an amplifier is to


(A) increase the input resistance and decrease the output resistance
(B) increases both input and output resistance
(C) decrease both input and output resistance
(D) decrease the input resistance and increase the output resistance

SOL 1.14

The effect of current shunt feedback in an amplifier is to decrease the input


resistance and increase the output resistance as :
Rif = Ri
1 + A
Rof = R0 (1 + A)
where Ri " Input resistance without feedback
Rif " Input resistance with feedback.
Hence (D) is correct option.

MCQ 1.15

SOL 1.15

The input resistance Ri of the amplifier shown in the figure is

(A) 30 k
4

(B) 10 k

(C) 40 k

(D) infinite

Since the inverting terminal is at virtual ground, the current flowing through the
voltage source is
Is = Vs
10k
Vs = 10 k = R
or
in
Is
Hence (B) is correct option.

Brought to you by: Nodia and Company


PUBLISHING FOR GATE

Visit us at: www.nodia.co.in

Page 7
MCQ 1.16

GATE EC 2005

The first and the last critical frequency of an RC -driving point impedance function
must respectively be
(A) a zero and a pole
(B) a zero and a zero
(C) a pole and a pole

SOL 1.16

www.gatehelp.com

(D) a pole and a zero

For stability poles and zero interlace on real axis. In RC series network the driving
point impedance is
Zins = R + 1 = 1 + sRC
Cs
sC
Here pole is at origin and zero is at s = 1/RC , therefore first critical frequency is
a pole and last critical frequency is a zero.
For RC parallel network the driving point impedance is
R 1
R
Cs =
Zinp =
1
1
+
sRC
R+
Cs
Here pole is s = 1/RC and zero is at 3, therefore first critical frequency is a pole
and last critical frequency is a zero.
Hence (C) is correct option.

MCQ 1.17

The cascade amplifier is a multistage configuration of


(A) CC CB
(B) CE CB
(C) CB CC

(D) CE CC

SOL 1.17

The CE configuration has high voltage gain as well as high current gain. It performs
basic function of amplifications. The CB configuration has lowest Ri and highest Ro
. It is used as last step to match a very low impedance source and to drain a high
impedance load
Thus cascade amplifier is a multistage configuration of CE-CB
Hence (B) is correct option

MCQ 1.18

Decimal 43 in Hexadecimal and BCD number system is respectively


(A) B2, 0100 011
(B) 2B, 0100 0011
(C) 2B, 0011 0100

SOL 1.18

(D) B2, 0100 0100

Dividing 43 by 16 we get
2
16 43
32
11
11 in decimal is equivalent is B in hexamal.
Thus
4310 * 2B16
Now
410 * 01002
310 * 00112
Thus
4310 * 01000011BCD

Brought to you by: Nodia and Company


PUBLISHING FOR GATE

Visit us at: www.nodia.co.in

Page 8

GATE EC 2005

www.gatehelp.com

Hence (B) is correct answer.


MCQ 1.19

SOL 1.19

The Boolean function f implemented in the figure using two input multiplexes is

(A) ABC + ABC

(B) ABC + ABC

(C) ABC + ABC

(D) ABC + ABC

The diagram is as shown in fig

f' = BC + BC
f = f' A + f ' 0
= f'A = ABC + ABC
Hence (A) is correct answer.
MCQ 1.20

Which of the following can be impulse response of a causal system ?

SOL 1.20

For causal system h (t) = 0 for t # 0 . Only (D) satisfy this condition.
Hence (D) is correct answer.

Brought to you by: Nodia and Company


PUBLISHING FOR GATE

Visit us at: www.nodia.co.in

Page 9

MCQ 1.21

GATE EC 2005

Let x (n) = ( 12 ) n u (n), y (n) = x2 (n) and Y (e j) be the Fourier transform of y (n) then
Y (e j0)
(A) 1
(B) 2
4
(D) 4
3

(C) 4
SOL 1.21

www.gatehelp.com

Hence (D) is correct answer.


n
x (n) = b 1 l u (n)
2
y (n) = x2 (n) = b 1 l u2 (n)
2
2n

or

2 n
n
y (n) = ;b 1 l E u (n) = b 1 l u (n)
2
4

Y (e j) =
or

Y (e j0) =

n=3

/ y (n) ejn

n = 3

Y (e j0) =

n=3

/ b 14 l ejn
n

n=0

n=3

/ ` 14 j

n=0

or

...(1)

1
1

1
4

1
3
4
= 1 +b1l +b1l+b1l +b1l
4
4
4
4
=4
3

Alternative :
Taking z transform of (1) we get
1
Y (z) =
1 14 z1
Substituting z = e j we have
1
Y (e j) =
1 14 ej
Y (e j0) = 1 1 = 4
3
1 4
MCQ 1.22

Find the correct match between group 1 and group 2.


Group 1
Group 2
W. Phase modulation
P. {1 + km (t) A sin (c t)}
Q. km (t) A sin (c t)
X. Frequency modulation
R. A sin {c t + km (t)}
Y. Amplitude modulation
t
S. A sin ; c t + k m (t) dt E
Z. DSB-SC modulation

#- 3

(A) P Z, Q Y, R X, S W
(B) P W, Q X, R Y, S Z
(C) P X, Q W, R Z, S Y
(D) P Y, Q Z, R W, S X
SOL 1.22

Hence (D) is correct option.


$ Amplitude modulation
{1 + km (t)} A sin (c t)

Brought to you by: Nodia and Company


PUBLISHING FOR GATE

Visit us at: www.nodia.co.in

Page 10

GATE EC 2005

www.gatehelp.com

dm (t) Asin (c t) $ DSB-SC modulation


A sin {cos t + km (t)} $ Phase Modulation
A sin [ct + k] t- 3 m (t) dt
$ Frequency Modulation
MCQ 1.23

The power in the signal s (t) = 8 cos (20 2 ) + 4 sin (15t) is


(A) 40
(B) 41
(C) 42

SOL 1.23

(D) 82

Hence (A) is correct answer.


s (t) = 8 cos ` 20t j + 4 sin 15t
2
= 8 sin 20t + 4 sin 15t
Here A1 = 8 and A2 = 4 . Thus power is
2
2
2
2
P = A1 + A2 = 8 + 4 = 40
2
2
2
2

MCQ 1.24

Which of the following analog modulation scheme requires the minimum transmitted
power and minimum channel bandwidth ?
(A) VSB
(B) DSB-SC
(C) SSB

(D) AM

SOL 1.24

Hence (C) is correct option.


VSB $ fm + fc
DSB - SC $ 2fm
SSB $ fm
AM $ 2fm
Thus SSB has minimum bandwidth and it require minimum power.

MCQ 1.25

A linear system is equivalently represented by two sets of state equations :


Xo = AX + BU and Wo = CW + DU
The eigenvalues of the representations are also computed as [] and []. Which one
of the following statements is true ?
(A) [] = [] and X = W
(B) [] = [] and X ! W
(C) [] ! [] and X = W

(D) [] = [] and X ! W

SOL 1.25

Hence (C) is correct option


We have Xo = AX + BU
where is set of Eigen values
o
and
where is set of Eigen values
W = CW + DU
If a liner system is equivalently represented by two sets of state equations, then for
both sets, states will be same but their sets of Eigne values will not be same i.e.
X = W but !

MCQ 1.26

Which one of the following polar diagrams corresponds to a lag network ?

Brought to you by: Nodia and Company


PUBLISHING FOR GATE

Visit us at: www.nodia.co.in

Page 11

SOL 1.26

GATE EC 2005

www.gatehelp.com

The transfer function of a lag network is


T (s) = 1 + sT
1 + sT

> 1; T > 0

1 + 2 T2
1 + 2 2 T2
and +T (j) = tan1 (T) tan1 (T)
At = 0 , T (j)
At = 0 ,+T (j)
T (j) =

=1
= tan1 0 = 0
= 1

At = 3 , T (j)
At = 3 ,+T (j)
Hence (D) is correct option.

=0

MCQ 1.27

Despite the presence of negative feedback, control systems still have problems of
instability because the
(A) Components used have non- linearities
(B) Dynamic equations of the subsystem are not known exactly.
(C) Mathematical analysis involves approximations.
(D) System has large negative phase angle at high frequencies.

SOL 1.27

Despite the presence of negative feedback, control systems still have problems
of instability because components used have nonlinearity. There are always some
variation as compared to ideal characteristics.
Hence (A) is correct option.

MCQ 1.28

The magnetic field intensity vector of a plane wave is given by


H (x, y, z, t) = 10 sin (50000t + 0.004x + 30) aty
t
where ay , denotes the unit vector in y direction. The wave is propagating with a
phase velocity.
(A) 5 # 10 4 m/s
(B) 3 # 108 m/s
(C) 1.25 # 107 m/s

Brought to you by: Nodia and Company


PUBLISHING FOR GATE

(D) 3 # 108 m/s]

Visit us at: www.nodia.co.in

Page 12

GATE EC 2005

www.gatehelp.com

SOL 1.28

Hence (C) is correct option.


= 50, 000 and = 0.004
4
Phase Velocity is vP = = 5 # 10 - 3 = 1.25 # 107 m/s

4 # 10

MCQ 1.29

Many circles are drawn in a Smith Chart used for transmission line calculations.
The circles shown in the figure represent

(A) Unit circles


(B) Constant resistance circles
(C) Constant reactance circles
(D) Constant reflection coefficient circles.
SOL 1.29

The given figure represent constant reactance circle.


Hence (C) is correct option.

MCQ 1.30

Refractive index of glass is 1.5. Find the wavelength of a beam of light with frequency
of 1014 Hz in glass. Assume velocity of light is 3 # 108 m/s in vacuum
(A) 3 m
(B) 3 mm
(C) 2 m

SOL 1.30

(D) 1 mm

Refractive index of glass = 1.5


Frequency f = 1014 Hz
c = 3 # 108 m/sec
8
= c = 3 # 10
= 3 # 10 - 6
14
f
10
wavelength in glass is
-6
g = = 3 # 10 = 2 # 10 - 6 m
1.5

Hence (C) is correct option.

Question 31 - 80 Carry Two Marks Each


MCQ 1.31

In what range should Re (s) remain so that the Laplace transform of the function
e(a + 2) t + 5 exits.
(A) Re (s) > a + 2
(B) Re (s) > a + 7

Brought to you by: Nodia and Company


PUBLISHING FOR GATE

Visit us at: www.nodia.co.in

Page 13

GATE EC 2005

(C) Re (s) < 2


SOL 1.31

MCQ 1.32

SOL 1.32

www.gatehelp.com

(D) Re (s) > a + 5

Hence (A) is correct answer.


We have f (t) = e(a + 2) t + 5 = e5 .e(a + 2) t
Taking laplace transform we get
1
F (s) = e5 ;
s (a + 2) E

Thus Re (s) > (a + 2)

4 2
Given the matrix =
, the eigenvector is
4 3G
3
4
(A) = G
(B) = G
2
3
1
(D) = G
2

2
(C) = G
1
Hence (C) is correct answer.
We have

4 2
A ==
4 3G

Characteristic equation is
A I = 0
4 2
or
=0
4 3
or
( 4 )(3 ) 8
or
12 + + 2 8
or
2 + 20
or

Eigen vector for = 5


(A I) Xi
1 ( 5) 2 x1
=
4
8 4G=x2 G
1 2 x1
=0 0G=x G
2

=0
=0
=0
= 5, 4

Eigen values

=0
0
== G
0
0
== G
0

R2 4R1

x1 + 2x2 = 0
Let x1 = 2 & x2 = 1,
Thus

MCQ 1.33

Let, A = =

X ==

2
1G

Eigen vector

1
2 0.1
a
and A - 1 = = 2 G. Then (a + b) =
G
0 3
0 b

(A) 7/20

(B) 3/20

(C) 19/60

(D) 11/20

Brought to you by: Nodia and Company


PUBLISHING FOR GATE

Visit us at: www.nodia.co.in

Page 14
SOL 1.33

GATE EC 2005

We have
A ==

www.gatehelp.com

1
2 0.1
a
and A - 1 = = 2 G
G
0 3
0 b

AA - 1 = I
2 0.1 12 a
1 0
G = =0 1 G
=0 3 G=
0 b
1 2a 0.1b
1 0
==
G
=0
3b
0 1G

Now
or
or

2a 0.1 = 0 and 3b = 1
Thus solving above we have b = 1 and a = 1
3
60
Therefore
a+b = 1 + 1 = 7
3 60
20
or

Hence (A) is correct option.


MCQ 1.34

SOL 1.34

The value of the integral I =

1
2

#0

exp c x m dx is
8

(A) 1

(B)

(C) 2

(D) 2

Gaussian PDF is
f (x) =
and

# f (x) dx
3

1
2

3 (x )2
22

dx

for 3 # x # 3

=1

Substituting = 0 and = 2 in above we get


3
1
e dx = 1
2 2 3
1 2 3e dx = 1
or
2 2 0
3
1
or
e dx = 1
2 0
Hence (A) is correct option.
MCQ 1.35

x2
8

x2
8

x2
8

The derivative of the symmetric function drawn in given figure will look like

Brought to you by: Nodia and Company


PUBLISHING FOR GATE

Visit us at: www.nodia.co.in

Page 15

GATE EC 2005

www.gatehelp.com

SOL 1.35

For x > 0 the slope of given curve is negative. Only (C) satisfy this condition.
Hence (C) is correct answer.

MCQ 1.36

Match the following and choose the correct combination:


Group I
Group 2
E. Newton-Raphson method
1. Solving nonlinear equations
F. Runge-kutta method
2. Solving linear simultaneous
equations
G. Simpsons Rule
3. Solving ordinary differential
equations
H. Gauss elimination
4. Numerical integration
5. Interpolation
6. Calculation of Eigenvalues
(B) E 1, F 6, G 4, H 3
(A) E 6, F 1, G 5, H 3
(C) E 1, F 3, G 4, H 2

SOL 1.36

MCQ 1.37

Hence (C) is correct option.


Newton - Raphson
"
Runge - kutta Method
"
Simpsons Rule
"
Gauss elimination
"

(D) E 5, F 3, G 4, H 1
Method-Solving nonlinear eq.
Solving ordinary differential eq.
Numerical Integration
Solving linear simultaneous eq.

Given an orthogonal matrix


R1 1 1 1 V
W
S
S1 1 1 1W
A =S
1 1 0 0W
W
S
S0 0 1 1 W
T
X
6AA @- 1 is T

V
R1
S 4 0 0 0W
S 0 14 0 0 W
W
(A) S
1
S0 0 2 0W
S 0 0 0 12 W
Brought to you by: Nodia
and Company
X
T
PUBLISHING FOR GATE

V
R1
S 2 0 0 0W
S 0 12 0 0 W
W
(B) S
1
S0 0 2 0 W
S 0 0 0 12 W
Visit
X us at: www.nodia.co.in
T

Page 16

GATE EC 2005

R1
S
S0
(C) S
0
S
S0
T

0
1
0
0

0
0
1
0

0 VW
0W
0W
W
1W
X

www.gatehelp.com

V
R1
S 4 0 0 0W
S0 1 0 0 W
(D) S 4 1 W
S0 0 4 0 W
S 0 0 0 14 W
X
T

SOL 1.37

Hence (C) is correct option.


From orthogonal matrix
[AAT ] = I
Since the inverse of I is I , thus
[AAT ] 1 = I1 = I

MCQ 1.38

For the circuit shown in the figure, the instantaneous current i1 (t) is

SOL 1.38

(A) 10 3 90c A
2

(B) 10 3 90c A
2

(C) 5 60c A

(D) 5 60c A

Applying KCL we get


i1 (t) + 5+0c = 10+60c
or
i1 (t) = 10+60c 5+0c = 5 + 5 3j 5
or
i1 (t) = 5 3 +90c = 10 3 +90c
2
Hence (A) is correct option.

MCQ 1.39

Impedance Z as shown in the given figure is

(A) j29

(B) j9

(C) j19

(D) j39

Brought to you by: Nodia and Company


PUBLISHING FOR GATE

Visit us at: www.nodia.co.in

Page 17

GATE EC 2005

www.gatehelp.com

SOL 1.39

If L1 = j5 and L3 = j2 the mutual induction is subtractive because current enters


from dotted terminal of j2 coil and exit from dotted terminal of j5. If L2 = j2
and L3 = j2 the mutual induction is additive because current enters from dotted
terminal of both coil.
Thus
Z = L1 M13 + L2 + M23 + L3 M31 + M32
= j5 + j10 + j2 + j10 + j2 j10 + j10 = j9
Hence (B) is correct option.

MCQ 1.40

For the circuit shown in the figure, Thevenins voltage and Thevenins equivalent
resistance at terminals a b is

SOL 1.40

(A) 5 V and 2

(B) 7.5 V and 2.5

(C) 4 V and 2

(D) 3 V and 2.5

Open circuit at terminal ab is shown below

Applying KCL at node we get


Vab + Vab 10 = 1
5
5
or
Vab = 7.5 = Vth
Short circuit at terminal ab is shown below

Short circuit current from terminal ab is


Isc = 1 + 10 = 3 A
5
Thus
Rth = Vth = 7.5 = 2.5
Isc
3
Brought to you by: Nodia and Company
PUBLISHING FOR GATE

Visit us at: www.nodia.co.in

Page 18

GATE EC 2005

www.gatehelp.com

Here current source being in series with dependent voltage source make it ineffective.
Hence (B) is correct option.
MCQ 1.41

SOL 1.41

If R1 = R2 = R4 = R and R3 = 1.1R in the bridge circuit shown in the figure, then


the reading in the ideal voltmeter connected between a and b is

(A) 0.238 V

(B) 0.138 V

(C) 0.238 V

(D) 1 V

Here Va = 5 V because R1 = R2 and total voltage drop is 10 V.


Now
Vb = R3 # 10 = 1.1 # 10 = 5.238 V
R3 + R4
2.1
V = Va Vb = 5 5.238 = 0.238 V
Hence (C) is correct option.

MCQ 1.42

The h parameters of the circuit shown in the figure are

0. 1 0. 1
(A) =
0. 1 0. 3 G

SOL 1.42

10 1
(B) =
1 0.05G

30 20
10 1
(C) =
(D) =
G
20 20
1 0.05G
For h parameters we have to write V1 and I2 in terms of I1 and V2 .
V1 = h11 I1 + h12 V2
I2 = h21 I1 + h22 V2
Applying KVL at input port
V1 = 10I1 + V2
Applying KCL at output port
V2 = I + I
1
2
20
or
I2 = I1 + V2
20

Brought to you by: Nodia and Company


PUBLISHING FOR GATE

Visit us at: www.nodia.co.in

Page 19

GATE EC 2005

www.gatehelp.com

Thus from above equation we get


h11 h12
10 1
=h h G = = 1 0.05G
12 22
Hence (D) is correct option.
MCQ 1.43

SOL 1.43

A square pulse of 3 volts amplitude is applied to C R circuit shown in the figure.


The capacitor is initially uncharged. The output voltage V2 at time t = 2 sec is

(A) 3 V

(B) 3 V

(C) 4 V

(D) 4 V

Hence (B) is correct option.


Time constant RC = 0.1 # 10 - 6 # 103 = 10 - 4 sec
Since time constant RC is very small, so steady state will be reached in 2 sec. At
t = 2 sec the circuit is as shown in fig.

Vc = 3 V
V2 = Vc = 3 V
MCQ 1.44

SOL 1.44

A Silicon sample A is doped with 1018 atoms/cm 3 of boron. Another sample B


of identical dimension is doped with 1018 atoms/cm 3 phosphorus. The ratio of
electron to hole mobility is 3. The ratio of conductivity of the sample A to B is
(A) 3
(B) 1
3
(C) 2
3
Hence option (B) is correct.
n = nqn
p = pqp
p

= p =1
n
n
3

Brought to you by: Nodia and Company


PUBLISHING FOR GATE

(D) 3
2

(n = p)

Visit us at: www.nodia.co.in

Page 20
MCQ 1.45

GATE EC 2005

www.gatehelp.com

A Silicon PN junction diode under reverse bias has depletion region of width 10
m. The relative permittivity of Silicon, r = 11.7 and the permittivity of free space
0 = 8.85 # 10 - 12 F/m. The depletion capacitance of the diode per square meter is
(A) 100 F
(B) 10 F
(C) 1 F

(D) 20 F

SOL 1.45

Hence option (B) is correct.


C = 0 r A
d
C = 0 r = 8.85 # 1012 # 11.7 = 10.35 F
or
d
A
10 # 106

MCQ 1.46

For an npn transistor connected as shown in figure VBE = 0.7 volts. Given that
reverse saturation current of the junction at room temperature 300 K is 10 - 13 A,
the emitter current is

SOL 1.46

(A) 30 mA

(B) 39 mA

(C) 49 mA

(D) 20 mA

Hence (C) is correct option.


V
IE = Is `e nV 1j = 10 - 13 c
BE

MCQ 1.47

0.7
1m = 49 mA
e1 # 26 # 10
-3

The voltage e0 is indicated in the figure has been measured by an ideal voltmeter.
Which of the following can be calculated ?

(A) Bias current of the inverting input only


(B) Bias current of the inverting and non-inverting inputs only
(C) Input offset current only
(D) Both the bias currents and the input offset current

Brought to you by: Nodia and Company


PUBLISHING FOR GATE

Visit us at: www.nodia.co.in

Page 21
SOL 1.47

GATE EC 2005

www.gatehelp.com

The circuit is as shown below

Writing equation for I have


e 0 V = I
1M
or
e0 = I (1M) + V
Writing equation for I+ we have
0 V+
= I+
1M

...(1)

or
...(2)
V+ = I+ (1M)
Since for ideal OPAMP V+ = V- , from (1) and (2) we have
e0 = I (1M) I + (1M)
= (I I+) (1M) = IOS (1M)
Thus if e0 has been measured, we can calculate input offset current IOS only.
Hence (C) is correct option.
MCQ 1.48

SOL 1.48

The Op-amp circuit shown in the figure is filter. The type of filter and its cut. Off
frequency are respectively

(A) high pass, 1000 rad/sec.

(B) Low pass, 1000 rad/sec

(C) high pass, 1000 rad/sec

(D) low pass, 10000 rad/sec

At low frequency capacitor is open circuit and voltage acr s non-inverting terminal
is zero. At high frequency capacitor act as short circuit and all input voltage appear
at non-inverting terminal. Thus, this is high pass circuit.
The frequency is given by
1
= 1000 rad/sec
= 1 =
RC
1 # 103 # 1 # 10 - 6
Hence (C) is correct option.

Brought to you by: Nodia and Company


PUBLISHING FOR GATE

Visit us at: www.nodia.co.in

Page 22
MCQ 1.49

GATE EC 2005

www.gatehelp.com

In an ideal differential amplifier shown in the figure, a large value of (RE ).


(A) increase both the differential and common - mode gains.
(B) increases the common mode gain only.
(C) decreases the differential mode gain only.
(D) decreases the common mode gain only.

SOL 1.49

Common mode gain


ACM = RC
2RE
And differential mode gain
ADM = gm RC
Thus only common mode gain depends on RE and for large value of RE it decreases.
Hence (D) is correct option.

MCQ 1.50

For an n -channel MOSFET and its transfer curve shown in the figure, the threshold
voltage is

(A) 1 V and the device is in active region


(B) 1 V and the device is in saturation region
(C) 1 V and the device is in saturation region
(D) 1 V and the device is an active region
SOL 1.50

From the graph it can be easily seen that Vth = 1 V


Now
VGS = 3 1 = 2 V
and
VDS = 5 1 = 4 V
Since
VDS > VGS $ VDS > VGS Vth
Thus MOSFET is in saturation region.
Hence option (C) is correct.

MCQ 1.51

The circuit using a BJT with = 50 and VBE = 0.7V is shown in the figure. The
base current IB and collector voltage by VC and respectively

Brought to you by: Nodia and Company


PUBLISHING FOR GATE

Visit us at: www.nodia.co.in

Page 23

SOL 1.51

GATE EC 2005

www.gatehelp.com

(A) 43 A and 11.4 Volts

(B) 40 A and 16 Volts

(C) 45 A and 11 Volts

(D) 50 A and 10 Volts

The circuit under DC condition is shown in fig below

Applying KVL we have


VCC RB IB VBE RE IE = 0
orVCC RB IB VBE RE ( + 1) IB = 0
or
IB = VCC VBE
RB + ( + 1) RE
20 0.7
=
= 40 A
430k + (50 + 1)1 k

Since IE = IB + IB

IC = IB = 50 # 40 = 2 mA
VC = VCC RC IC = 20 2m # 2k = 16 V
Hence (B) is correct option.

Now

MCQ 1.52

The Zener diode in the regulator circuit shown in the figure has a Zener voltage of
5.8 volts and a zener knee current of 0.5 mA. The maximum load current drawn
from this current ensuring proper functioning over the input voltage range between
20 and 30 volts, is

Brought to you by: Nodia and Company


PUBLISHING FOR GATE

Visit us at: www.nodia.co.in

Page 24

SOL 1.52

GATE EC 2005

www.gatehelp.com

(A) 23.7 mA

(B) 14.2 mA

(C) 13.7 mA

(D) 24.2 mA

The maximum load current will be at maximum input voltage i.e.


Vmax = 30 V i.e.
Vmax VZ = I + I
L
Z
1k
or 30 5.8 = IL = 0.5 m
1k
or
IL = 24.2 0.5 = 23.7 mA
Hence (A) is correct option.

MCQ 1.53

SOL 1.53

The transistors used in a portion of the TTL gate show in the figure have = 100 .
The base emitter voltage of is 0.7 V for a transistor in active region and 0.75 V for
a transistor in saturation. If the sink current I = 1 A and the output is at logic 0,
then the current IR will be equal to

(A) 0.65 mA

(B) 0.70 mA

(C) 0.75 mA

(D) 1.00 mA

The circuit is as shown below

If output is at logic 0, the we have V0 = 0 which signifies BJT Q3 is in saturation


and applying KVL we have
VBE3 = IR # 1k
or
0.75 = IR # 1k
or
IR = 0.75 mA
Brought to you by: Nodia and Company
PUBLISHING FOR GATE

Visit us at: www.nodia.co.in

Page 25

GATE EC 2005

www.gatehelp.com

Hence (C) is correct answer.


MCQ 1.54

The Boolean expression for the truth table shown is

(A) B (A + C)( A + C )

(B) B (A + C )( A + C)

(C) B (A + C )( A + C)

(D) B (A + C)( A + C )

SOL 1.54

Hence (A) is correct answer.


We have
f = ABC + ABC
= B (AC + AC ) = B (A + C)( A + C )

MCQ 1.55

Both transistors T1 and T2 show in the figure, have a = 100 , threshold voltage of
1 Volts. The device parameters K1 and K2 of T1 and T2 are, respectively, 36 A/V2
and 9 A/V 2 . The output voltage Vo i s

(A) 1 V

(B) 2 V

(C) 3 V

(D) 4 V

SOL 1.55

Hence (D) is correct option.

MCQ 1.56

The present output Qn of an edge triggered JK flip-flop is logic 0. If J = 1, then


Qn + 1
(A) Cannot be determined
(B) Will be logic 0
(C) will be logic 1

SOL 1.56

(D) will rave around

Characteristic equation for a jk flip-flop is written as


Qn + 1 = JQ n + K Qn
Where
Qn is the present output
Qn + 1 is next output
So,
Qn + 1 = 10 + K : 0

Brought to you by: Nodia and Company


PUBLISHING FOR GATE

Qn = 0
Visit us at: www.nodia.co.in

Page 26

GATE EC 2005

www.gatehelp.com

Qn + 1 = 1
Hence (C) is correct answer.
MCQ 1.57

The given figure shows a ripple counter using positive edge triggered flip-flops. If
the present state of the counter is Q2 Q1 Q0 = 001 then is next state Q2 Q1 Q will be

(A) 010

(B) 111

(C) 100

(D) 101

SOL 1.57

Since T2 T1 T0 is at 111, at every clock Q2 Q1 Q0 will be changes. Ir present state is


011, the next state will be 100.
Hence (C) is correct answer.

MCQ 1.58

What memory address range is NOT represents by chip # 1 and chip # 2 in the
figure A0 to A15 in this figure are the address lines and CS means chip select.

(A) 0100 - 02FF

(B) 1500 - 16FF

(C) F900 - FAFF

(D) F800 - F9FF

SOL 1.58

Hence (D) is correct answer.

MCQ 1.59

The output y (t) of a linear time invariant system is related to its input x (t) by the
following equations
y (t)= 0.5x (t td + T) + x (t td ) + 0.5x (t td + T)

Brought to you by: Nodia and Company


PUBLISHING FOR GATE

Visit us at: www.nodia.co.in

Page 27

GATE EC 2005

www.gatehelp.com

The filter transfer function H () of such a system is given by


(A) (1 + cos T) ejt
(B) (1 + 0.5 cos T) ejt
d

(C) (1 cos T) ejt

(D) (1 0.5 cos T) ejt

SOL 1.59

Hence (A) is correct answer.


y (t) = 0.5x (t td + T) + x (t td ) + 0.5x (t td T)
Taking Fourier transform we have
Y () = 0.5ej ( t + T) X () + ejt X () + 0.5ej ( t T) X ()
d

or

Y ()
= ejt [0.5e jT + 1 + 0.5ejT ]
X ()
d

= ejt [0.5 (e jT + ejT ) + 1] = ejt [cos T + 1]


d

or
MCQ 1.60

H () =

Y ()
= ejt (cos T + 1)
X ()
d

Match the following and choose the correct combination.


Group 1
E. Continuous and aperiodic signal
F. Continuous and periodic signal
G. Discrete and aperiodic signal
H. Discrete and periodic signal
Group 2
1. Fourier representation is continuous and aperiodic
2. Fourier representation is discrete and aperiodic
3. Fourier representation is continuous and periodic
4. Fourier representation is discrete and periodic
(A) E 3, F 2, G 4, H 1
(B) E 1, F 3, G 2, H 4
(C) E 1, F 2, G 3, H 4
(D) E 2, F 1, G 4, H 3

SOL 1.60

For continuous and aperiodic signal Fourier representation is continuous and


aperiodic.
For continuous and periodic signal Fourier representation is discrete and aperiodic.
For discrete and aperiodic signal Fourier representation is continuous and periodic.
For discrete and periodic signal Fourier representation is discrete and periodic.
Hence (C) is correct answer.

MCQ 1.61

A signal x (n) = sin (0 n + ) is the input to a linear time- invariant system having
a frequency response H (e j). If the output of the system Ax (n n0) then the most
general form of +H (e j) will be

Brought to you by: Nodia and Company


PUBLISHING FOR GATE

Visit us at: www.nodia.co.in

Page 28

GATE EC 2005

www.gatehelp.com

(A) n0 0 + for any arbitrary real


(B) n0 0 + 2k for any arbitrary integer k
(C) n0 0 + 2k for any arbitrary integer k
(D) n0 0
SOL 1.61

Hence (B) is correct answer.


y (n) = Ax (n no)
Taking Fourier transform
Y (e j) = Aej n X (e j)
Y (e j)
j n
or
H (e j) =
j = Ae
X (e )
j
Thus +H (e ) = o no
For LTI discrete time system phase and frequency of H (e j) are periodic with
period 2. So in general form
() = no o + 2k
o

MCQ 1.62

For a signal x (t) the Fourier transform is X (f). Then the inverse Fourier transform
of X (3f + 2) is given by
j4t
(A) 1 x` t j e j3t
(B) 1 x` t j e - 3
2 2
3 3
(C) 3x (3t) ej4t

SOL 1.62

(D) x (3t + 2)

Hence (B) is correct answer.


F
x (t)
X (f)
Using scaling we have
F
1 X f
x (at)
a ca m
F
Thus x b 1 f l
3X (3f)
3
Using shifting property we get
ej2f t x (t) = X (f + f0)
4
Thus 1 ej 3 t x b 1 t l
3
3
F
ej2 t x b 1 t l
3X (3 (f + 23 ))
3
F
1 ej t x 1 t
X [3 (f + 23 )]
b3 l
3
0

X (3f + 2)

2
3

4
3

MCQ 1.63

The polar diagram of a conditionally stable system for open loop gain K = 1 is
shown in the figure. The open loop transfer function of the system is known to be
stable. The closed loop system is stable for

Brought to you by: Nodia and Company


PUBLISHING FOR GATE

Visit us at: www.nodia.co.in

Page 29

SOL 1.63
MCQ 1.64

GATE EC 2005

www.gatehelp.com

(A) K < 5 and 1 < K < 1


2
8

(B) K < 1 and 1 < K < 5


8
2

(C) K < 1 and 5 < K


8
Hence (B) is correct option

(D) K > 1 and 5 > K


8

In the derivation of expression for peak percent overshoot



Mp = exp e
o # 100%
1 2
Which one of the following conditions is NOT required ?
(A) System is linear and time invariant
(B) The system transfer function has a pair of complex conjugate poles and no
zeroes.
(C) There is no transportation delay in the system.
(D) The system has zero initial conditions.

SOL 1.64

The peak percent overshoot is determined for LTI second order closed loop system
with zero initial condition. Its transfer function is
n2
T (s) = 2
s + 2n s + n2
Transfer function has a pair of complex conjugate poles and zeroes.
Hence (C) is correct option.

MCQ 1.65

Given the ideal operational amplifier circuit shown in the figure indicate the correct
transfer characteristics assuming ideal diodes with zero cut-in voltage.

Brought to you by: Nodia and Company


PUBLISHING FOR GATE

Visit us at: www.nodia.co.in

Page 30

SOL 1.65

GATE EC 2005

www.gatehelp.com

Only one diode will be in ON conditions


When lower diode is in ON condition, then
Vu = 2k Vsat = 2 10 = 8 V
2.5k
2.5
when upper diode is in ON condition
Vu = 2k Vsat = 2 ( 10) = 5 V
2.5k
4
Hence (B) is correct option.

MCQ 1.66

SOL 1.66

A ramp input applied to an unity feedback system results in 5% steady state error.
The type number and zero frequency gain of the system are respectively
(A) 1 and 20
(B) 0 and 20
(C) 0 and 1
(D) 1 and 1
20
20
1
For ramp input we have R (s) = 2
s
Now
ess = lim sE (s)
s"0

Brought to you by: Nodia and Company


PUBLISHING FOR GATE

Visit us at: www.nodia.co.in

Page 31

GATE EC 2005

www.gatehelp.com

R (s)
1
= lim
1 + G (s) s " 0 s + sG (s)
or
ess = lim 1 = 5% = 1 Finite
s " 0 sG (s)
20
But kv = 1 = lim sG (s) = 20
s"0
ess
= lim s
s"0

kv is finite for type 1 system having ramp input.


Hence (A) is correct option.
MCQ 1.67

A double integrator plant G (s) = K/s2, H (s) = 1 is to be compensated to achieve


the damping ratio = 0.5 and an undamped natural frequency, n = 5 rad/sec
which one of the following compensator Ge (s) will be suitable ?
(A) s + 3
(B) s + 99
s + 99
s+3
s6
s + 8.33
Hence (A) is correct option.
(C)

SOL 1.67

(D) s 6
s

MCQ 1.68

K (1 s)
An unity feedback system is given as G (s) =
.
s (s + 3)
Indicate the correct root locus diagram.

SOL 1.68

Hence (A) is correct option.

MCQ 1.69

A MOS capacitor made using p type substrate is in the accumulation mode. The
dominant charge in the channel is due to the presence of
(A) holes
(B) electrons
(C) positively charged icons

SOL 1.69

(D) negatively charged ions

In accumulation mode for NMOS having p substrate, when positive voltage is


applied at the gate, this will induce negative charge near p type surface beneath
the gate. When VGS is made sufficiently large, an inversion of electrons is formed
and this in effect forms and n channel.
Hence option (B) is correct.

Brought to you by: Nodia and Company


PUBLISHING FOR GATE

Visit us at: www.nodia.co.in

Page 32
MCQ 1.70

GATE EC 2005

A device with input X (t) and output y (t) is characterized by: Y (t) = x2 (t). An FM
signal with frequency deviation of 90 kHz and modulating signal bandwidth of 5
kHz is applied to this device. The bandwidth of the output signal is
(A) 370 kHz
(B) 190 kHz
(C) 380 kHz

SOL 1.70

www.gatehelp.com

(D) 95 kHz

Let x (t) be the input signal where


x (t) = cos (cos t + 1 cos m t)
cos (2c t + 21 cos m t)
y (t) = x2 (t) = 1 +
2
2
3f
Here
= 21 and 1 =
= 90 = 18
fm
5
BW = 2 ( + 1) fm = 2 (2 # 18 + 1) # 5 = 370 kHz
Hence (A) is correct option.

MCQ 1.71

A signal as shown in the figure is applied to a matched filter. Which of the following
does represent the output of this matched filter ?

SOL 1.71

The transfer function of matched filter is


h (t) = x (t t) = x (2 t)
The output of matched filter is the convolution of x (t) and h (t) as shown below

Brought to you by: Nodia and Company


PUBLISHING FOR GATE

Visit us at: www.nodia.co.in

Page 33

GATE EC 2005

www.gatehelp.com

Hence (C) is correct option.


MCQ 1.72

Noise with uniform power spectral density of N0 W/Hz is passed though a filter
H () = 2 exp ( jtd ) followed by an ideal pass filter of bandwidth B Hz. The
output noise power in Watts is
(A) 2N0 B
(B) 4N0 B
(C) 8N0 B

SOL 1.72

(D) 16N0 B

Hence (B) is correct option.


We have H (f) = 2e - jt
H (f) = 2
G0 (f) = H (f) 2 Gi (f)
= 4No W/Hz
The noise power is
d

MCQ 1.73

A carrier is phase modulated (PM) with frequency deviation of 10 kHz by a single


tone frequency of 1 kHz. If the single tone frequency is increased to 2 kHz, assuming
that phase deviation remains unchanged, the bandwidth of the PM signal is
(A) 21 kHz
(B) 22 kHz
(C) 42 kHz

SOL 1.73

= 4No # B

(D) 44 kHz

The phase deviation is


3f
=
= 10 = 10
fm
1
If phase deviation remain same and modulating frequency is changed
BW = 2 ( + 1) fm' = 2 (10 + 1) 2 = 44 kHz
Hence (D) is correct option.

MCQ 1.74

An output of a communication channel is a random variable v with the probability


density function as shown in the figure. The mean square value of v is

Brought to you by: Nodia and Company


PUBLISHING FOR GATE

Visit us at: www.nodia.co.in

Page 34

SOL 1.74

GATE EC 2005

www.gatehelp.com

(A) 4

(B) 6

(C) 8

(D) 9

As the area under pdf curve must be unity


1 (4 # k) = 1 $ k = 1
2
2
Now mean square value is
v2 =

#- 3

+3

v2 p (v) dv

#0

v2 ` v j dv
8

#0

v3
c 8 m dv = 8

as p (v) = 1 v
8

Hence (C) is correct option.


MCQ 1.75

Which one of the following does represent the electric field lines for the mode in the
cross-section of a hollow rectangular metallic waveguide ?

SOL 1.75

Hence (D) is correct option.

MCQ 1.76

Characteristic impedance of a transmission line is 50 . Input impedance of the


open-circuited line when the transmission line a short circuited, then value of the
input impedance will be.
(A) 50
(B) 100 + j150
(C) 7.69 + j11.54

SOL 1.76

(D) 7.69 j11.54

Hence (D) is correct option


Zo2 = ZOC .ZSC
2
ZZC = Zo = 50 # 50 = 50
ZOC
100 + j150
2 + 3j

Brought to you by: Nodia and Company


PUBLISHING FOR GATE

Visit us at: www.nodia.co.in

Page 35

GATE EC 2005

www.gatehelp.com

50 (2 3j)
= 7.69 11.54j
13

Hence (D) is correct option


MCQ 1.77

SOL 1.77

Two identical and parallel dipole antennas are kept apart by a distance of in the
4
H - plane. They are fed with equal currents but the right most antenna has a phase
shift of + 90c. The radiation pattern is given as.

The array factor is


A = cos b

d sin +
l
2

Here = 2 , d = and = 90c

4
2
sin + 2

Thus
A = cos c 4
m = cos ` sin + j
2
4
2
The option (A) satisfy this equation.

Common Data Questions 78, 79 and 80 :


Given, rd = 20k , IDSS = 10 mA, Vp = 8 V

Brought to you by: Nodia and Company


PUBLISHING FOR GATE

Visit us at: www.nodia.co.in

Page 36
MCQ 1.78

GATE EC 2005

www.gatehelp.com

Zi and Z0 of the circuit are respectively


(A) 2 M and 2 k

(B) 2 M and 20 k
11

(D) infinity and 20 k


11
The small signal model is as shown below
(C) infinity and 2 M

SOL 1.78

From the figure we have


Zin = 2 M
and

Z0 = rd RD = 20k 2k = 20 k
11

Hence (B) is correct option.


MCQ 1.79

ID and VDS under DC conditions are respectively


(A) 5.625 mA and 8.75 V
(B) 1.875 mA and 5.00 V
(C) 4.500 mA and 11.00 V

SOL 1.79

(D) 6.250 mA and 7.50 V

The circuit in DC condition is shown below

Since the FET has high input resistance, gate current can be neglect and we get
VGS = 2 V
Since VP < VGS < 0 , FET is operating in active region
2
( 2) 2
Now
ID = IDSS c1 VGS m = 10 c1
= 5.625 mA
( 8) m
VP
Now
VDS = VDD ID RD = 20 5.625 m # 2 k = 8.75 V
Hence (A) is correct option.
MCQ 1.80

Transconductance in milli-Siemens (mS) and voltage gain of the amplifier are

Brought to you by: Nodia and Company


PUBLISHING FOR GATE

Visit us at: www.nodia.co.in

Page 37

SOL 1.80

GATE EC 2005

www.gatehelp.com

respectively
(A) 1.875 mS and 3.41

(B) 1.875 ms and -3.41

(C) 3.3 mS and -6

(D) 3.3 mS and 6

The transconductance is
2
gm =
VP ID IDSS
or,
= 2 5.625mA # 10mA = 1.875 mS
8
The gain is A = gm (rd RD)
So,
= 1.875ms # 20 K = 3.41
11
Hence (B) is correct option.

Linked Answer Questions : Q. 81 to 90 Carry Two Marks Each


Consider an 8085 microprocessor system.
MCQ 1.81

The following program starts at location 0100H.


LXI SP, OOFF
LXI H, 0701
MVI A, 20H
SUB M
The content of accumulator when the program counter reaches 0109 H is
(A) 20 H
(B) 02 H
(C) 00 H

(D) FF H

SOL 1.81

0100H LXI SP, 00FF


0103H LXI H, 0701
0106H MVI A, 20H
0108 H SUB M

; Load SP with 00FFG


; Load HL with 0107H
; Move A with 20 H
; Subtract the contents of memory
; location whose address is stored in HL
; from the A and store in A
0109H ORI 40H
; 40H OR [A] and store in A
010BH ADD M
; Add the contents of memeory location
; whose address is stored in HL to A
; and store in A
HL contains 0107H and contents of 0107H is 20H
Thus after execution of SUB the data of A is 20H - 20H = 00
Hence (C) is correct answer.

MCQ 1.82

If in addition following code exists from 019H onwards,


ORI 40 H
ADD M

Brought to you by: Nodia and Company


PUBLISHING FOR GATE

Visit us at: www.nodia.co.in

Page 38

GATE EC 2005

www.gatehelp.com

What will be the result in the accumulator after the last instruction is executed ?
(A) 40 H
(B) 20 H
(C) 60 H
SOL 1.82

(D) 42 H

Before ORI instruction the contents of A is 00H. On execution the ORI 40H the
contents of A will be 40H
00H = 00000000
40H = 01000000
ORI 01000000
After ADD instruction the contents of memory location whose address is stored in
HL will be added to and will be stored in A
40H + 20 H = 60 H
Hence (C) is correct answer.

Statement for Linked Answer Question 83 and 84 :

MCQ 1.83

The open loop transfer function of a unity feedback system is given by


2s
G (s) = 3e
s (s + 2)
The gain and phase crossover frequencies in rad/sec are, respectively
(A) 0.632 and 1.26
(B) 0.632 and 0.485
(C) 0.485 and 0.632

SOL 1.83

(D) 1.26 and 0.632

Hence (D) is correct option


2s
G (s) = 3e
s (s + 2)
3e2j
j (j + 2)
3
G (j) =
2 + 4
Let at frequency g the gain is 1. Thus
3
=1
g (g2 + 4)
or

G (j) =

g4 + 4g2 9 = 0
g2 = 1.606
g = 1.26 rad/sec
Now +G (j) = 2 tan1
2
2

or
or
or

Let at frequency we have +GH = 180c

= 2 tan1
2
2

or
2 + tan1 =
2
2
Brought to you by: Nodia and Company
PUBLISHING FOR GATE

Visit us at: www.nodia.co.in

Page 39

GATE EC 2005

or

2 + c

1 3
` jm =
2
2
3 2

or

5 3
=

2
2
24
5
.
2
2

or
MCQ 1.84

= 0.63 rad/sec

Based on the above results, the gain and phase margins of the system will be
(A) 7.09 dB and 87.5c
(B) 7.09 dB and 87.5c
(C) 7.09 dB and 87.5c

SOL 1.84

www.gatehelp.com

(D) 7.09 and 87.5c

The gain at phase crossover frequency is


3
3
G (jg) =
=
2
( + 4)
0.63 (0.632 + 4)
or
G (jg) = 2.27
G.M. = 20 log G (jg)
20 log 2.26 = 7.08 dB
Since G.M. is negative system is unstable.
The phase at gain corss over frequency is

+G (jg) = 2g tan1 g
2
2
= 2 # 1.26 tan1 1.26
2
2

1
2

= 4.65 rad or 266.5c


PM = 180c + +G (jg) = 180c 266.5c = 86.5c
Hence (D) is correct option.
or

Common Data for Question 85 and 86 :


Asymmetric three-level midtread quantizer is to be designed assuming equiprobable
occurrence of all quantization levels.

MCQ 1.85

If the probability density function is divide into three regions as shown in the
figure, the value of a in the figure is
(A) 1
(B) 2
3
3

Brought to you by: Nodia and Company


PUBLISHING FOR GATE

Visit us at: www.nodia.co.in

Page 40

SOL 1.85

GATE EC 2005

www.gatehelp.com

(C) 1
(D) 1
2
4
As the area under pdf curve must be unity and all three region are equivaprobable.
Thus are under each region must be 13 .
2a # 1 = 1 $ a = 2
4
3
3
Hence (B) is correct option.

MCQ 1.86

SOL 1.86

The quantization noise power for the quantization region between a and + a in
the figure is
(A) 4
(B) 1
81
9
(C) 5
81
Hence (A) is correct option.
Nq =

#- a

+a

x2 p (x) dx = 2

(D) 2
81

#0

3 a
3
x $ 1 dx = 1 ; x E = a
4
2 3 0
6

a 2

Substituting a = 2 we have
3
Nq = 4
81

Statement of Linked Answer Questions 87 & 88 :


Voltage standing wave pattern in a lossless transmission line with characteristic
impedance 50 and a resistive load is shown in the figure.

MCQ 1.87

SOL 1.87

The value of the load resistance is


(A) 50

(B) 200

(C) 12.5

(D) 0

From the diagram, VSWR is


s = Vmax = 4 = 4
Vmin
1
When minima is at load ZO = s.ZL
or
ZL = Zo = 50 = 12.5
s
4

Brought to you by: Nodia and Company


PUBLISHING FOR GATE

Visit us at: www.nodia.co.in

Page 41

GATE EC 2005

www.gatehelp.com

Hence (C) is correct option.


MCQ 1.88

SOL 1.88

reflection coefficient is given by


(A) 0.6

(B) 1

(C) 0.6

(D) 0

The reflection coefficient is


= ZL ZO = 12.5 50 = 0.6
ZL + ZO
125. + 50
Hence (A) is correct option.

Statement of linked answer question 89 and89:


A sequence x (n) has non-zero values as shown in the figure.

x ( n2 1),

MCQ 1.89

The sequence y (n) = *


0,

Brought to you by: Nodia and Company


PUBLISHING FOR GATE

For n even
will be
For n odd

Visit us at: www.nodia.co.in

Page 42

SOL 1.89

MCQ 1.90

GATE EC 2005

www.gatehelp.com

Thus (A) is correct option.


From
x (n) = [ 12 , 1, 2, 1, 1, 12 ]
y (n) = x ^ n2 1h, n even
= 0 , for n odd
n = 2 ,
y ( 2) = x ( 22 1) = x ( 2) = 12
n = 1,
y ( 1) = 0
n = 0,
y (0) = x ( 20 1) = x ( 1) = 1
n = 1,
y (1) = 0
n=2
y (2) = x ( 22 1) = x (0) = 2
n = 3,
y (3) = 0
n=4
y (4) = x ( 24 1) = x (1) = 1
n = 5,
y (5) = 0
n=6
y (6) = x ( 26 1) = x (2) = 12
Hence
y (n) = 1 (n + 2) + (n) + 2 (n 2) + (n 4)+ 1 (n 6)
2
2
The Fourier transform of y (2n) will be
(A) ej2 [cos 4 + 2 cos 2 + 2]
(B) cos 2 + 2 cos + 2

Brought to you by: Nodia and Company


PUBLISHING FOR GATE

Visit us at: www.nodia.co.in

Page 43

GATE EC 2005

(C) ej [cos 2 + 2 cos + 2]


SOL 1.90

www.gatehelp.com

(D) ej2 [cos 2 + 2 cos + 2]

Here y (n) is scaled and shifted version of x (n) and again y (2n) is scaled version of
y (n) giving
z (n) = y (2n) = x (n 1)
= 1 (n + 1) + (n) + 2 (n 1) + (n 2) + 1 (n 3)
2
2
Taking Fourier transform.
Z (e j) = 1 e j + 1 + 2ej + e2j + 1 e3j
2
2
= ej b 1 e2j + e j + 2 + ej + 1 e2j l
2
2
= ej b e

2j

+ e2j + e j + 2 + ej
l
2

or
Z (e j) = ej [cos 2 + 2 cos + 2]
Hence (C) is correct answer

Answer Sheet

1.

(B)

19.

(A)

37.

(C)

55.

(D)

73.

(D)

2.

(C)

20.

(D)

38.

(A)

56.

(C)

74.

(C)

3.

(D)

21.

(D)

39.

(B)

57.

(C)

75.

(D)

4.

(B)

22.

(D)

40.

(B)

58.

(D)

76.

(D)

5.

(A)

23.

(A)

41.

(C)

59.

(A)

77.

(A)

6.

(C)

24.

(C)

42.

(D)

60.

(C)

78.

(B)

7.

(C)

25.

(C)

43.

(B)

61.

(B)

79.

(A)

8.

(B)

26.

(D)

44.

(B)

62.

(B)

80.

(B)

9.

(B)

27.

(A)

45.

(B)

63.

(B)

81.

(C)

10.

(C)

28.

(C)

46.

(C)

64.

(C)

82.

(C)

11.

(C)

29.

(C)

47.

(C)

65.

(B)

83.

(D)

12.

(B)

30.

(C)

48.

(C)

66.

(A)

84.

(D)

13.

(A)

31.

(A)

49.

(D)

67.

(A)

85.

(B)

14.

(D)

32.

(C)

50.

(C)

68.

(A)

86.

(A)

15.

(B)

33.

(A)

51.

(B)

69.

(B)

87.

(C)

16.

(C)

34.

(A)

52.

(A)

70

(A)

88.

(A)

17.

(B)

35.

(C)

53.

(C)

71

(C)

89.

(A)

18.

(B)

36.

(C)

54.

(A)

72

(B)

90.

(C)

Brought to you by: Nodia and Company


PUBLISHING FOR GATE

Visit us at: www.nodia.co.in

You might also like